www.vorhilfe.de
- Förderverein -
Der Förderverein.

Gemeinnütziger Verein zur Finanzierung des Projekts Vorhilfe.de.
Hallo Gast!einloggen | registrieren ]
Startseite · Mitglieder · Impressum
Forenbaum
^ Forenbaum
Status VH e.V.
  Status Vereinsforum

Gezeigt werden alle Foren bis zur Tiefe 2

Navigation
 Startseite...
 Suchen
 Impressum
Das Projekt
Server und Internetanbindung werden durch Spenden finanziert.
Organisiert wird das Projekt von unserem Koordinatorenteam.
Hunderte Mitglieder helfen ehrenamtlich in unseren moderierten Foren.
Anbieter der Seite ist der gemeinnützige Verein "Vorhilfe.de e.V.".
Partnerseiten
Weitere Fächer:

Open Source FunktionenplotterFunkyPlot: Kostenloser und quelloffener Funktionenplotter für Linux und andere Betriebssysteme
Forum "Uni-Analysis-Sonstiges" - Basis bestimmen
Basis bestimmen < Sonstiges < Analysis < Hochschule < Mathe < Vorhilfe
Ansicht: [ geschachtelt ] | ^ Forum "Uni-Analysis-Sonstiges"  | ^^ Alle Foren  | ^ Forenbaum  | Materialien

Basis bestimmen: Frage (beantwortet)
Status: (Frage) beantwortet Status 
Datum: 10:26 So 10.07.2016
Autor: JXner

Aufgabe
Wir betrachten den reellen Vektorraum P der reellen Polynome des Grades ≤ 2 zusammen mitder Addition von Polynomen (p+q)(x) :=p(x) + q(x) und der Multiplikation mit Konstanten (c·p)(x) := c·p(x). Wir setzen P₁(x) := x² + x + 1 p₂(x) := x –1 p₃(x) := x² -3x+2 Ist B (p₁, p₂, p₃) eine Basis von (P, +, ·)?

Zu  dieser Aufgabe gibt es folgende Lösung:

a₁ + a₂ + a₃= 0
a₂ -a₃= 0
a₁ -3a₂ + 2a₃= 0
a₁, a₂, a₃= 0   -> Die Vektoren sind Linear unabhängig

Koeffizientenvergleich
a₁ + a₂ + a₃= s
a₂ -a₃= t
a₁ -3a₂ + 2
a₃= u

a₁= s + t –(s-u)/4
a₂= (s-u)/4
a₃= (s-u)/4 –t

Antwort: Daraus folgt, dass das p1, p2, p3 keine Basis bilden.


Bisher kann ich alles soweit nachvollziehen, jedoch fehlt es mir an Verständnis beim Koeffizientenvergleich.
Warum bilden p1, p2, p3 nun keine Basis?
Wie müsste das Ergebnis aussehen wenn es eine Basis wäre?

        
Bezug
Basis bestimmen: Antwort
Status: (Antwort) fertig Status 
Datum: 14:05 So 10.07.2016
Autor: hippias


> Wir betrachten den reellen Vektorraum P der reellen
> Polynome des Grades ≤ 2 zusammen mitder Addition von
> Polynomen (p+q)(x) :=p(x) + q(x) und der Multiplikation mit
> Konstanten (c·p)(x) := c·p(x). Wir setzen P₁(x) := x²
> + x + 1 p₂(x) := x –1 p₃(x) := x² -3x+2 Ist B (p₁,
> p₂, p₃) eine Basis von (P, +, ·)?
>  Zu  dieser Aufgabe gibt es folgende Lösung:
>  
> a₁ + a₂ + a₃= 0
>  a₂ -a₃= 0
>  a₁ -3a₂ + 2a₃= 0
>  a₁, a₂, a₃= 0   -> Die Vektoren sind Linear

> unabhängig

Ich sehe keinen Zusammenhang zur Aufgabenstellung: was sind die [mm] $a_{i}$? [/mm]

>  
> Koeffizientenvergleich
>  a₁ + a₂ + a₃= s
>  a₂ -a₃= t
>  a₁ -3a₂ + 2
>  a₃= u

Was soll das jetzt? Was sind $s$, $t$ und $u$? Oben waren die rechten Seiten doch noch $=0$.

>  
> a₁= s + t –(s-u)/4
>  a₂= (s-u)/4
>  a₃= (s-u)/4 –t

Offenbar wurde das zweite Gleichungssystem nach den [mm] $a_{i}$ [/mm] aufgelöst.

>  
> Antwort: Daraus folgt, dass das p1, p2, p3 keine Basis
> bilden.
>  
>
> Bisher kann ich alles soweit nachvollziehen, jedoch fehlt

Mir ist nichts klar. Könntest Du die Lösung bitte erläutern?

> es mir an Verständnis beim Koeffizientenvergleich.
>  Warum bilden p1, p2, p3 nun keine Basis?
>  Wie müsste das Ergebnis aussehen wenn es eine Basis
> wäre?

Keine Ahnung! Wenn Du weisst, dass der obige Raum $3$ dimensional ist, dann genügt es zu überprüfen, ob die Polynome [mm] $p_{1}$, $p_{2}$ [/mm] und [mm] $p_{3}$ [/mm] linear unabhängig sind. Seien dazu [mm] $c_{i}\in \IR$ [/mm] so, dass [mm] $\sum_{i=1}^{3} c_{i}p_{i}=0$ [/mm] ist. Ordne die linke Seite nach Potenzen von $x$, mache den Koeffizientenvergleich und löse das entsprechende Gleichungssystem auf.


Bezug
                
Bezug
Basis bestimmen: Frage (reagiert)
Status: (Frage) reagiert/warte auf Reaktion Status 
Datum: 17:28 So 10.07.2016
Autor: JXner


> Ordne die linke Seite nach Potenzen von [mm]x[/mm], mache den
> Koeffizientenvergleich und löse das entsprechende
> Gleichungssystem auf.
>    

Meine Frage bei dieser Aufgabe war, wie man ein Koeffizientenvergleich durchführt. Mit "mache den Koeffizientenvergleich" kann ich daher nicht viel anfangen.

Bezug
        
Bezug
Basis bestimmen: Antwort
Status: (Antwort) fertig Status 
Datum: 21:13 So 10.07.2016
Autor: meili

Hallo,

> Wir betrachten den reellen Vektorraum P der reellen
> Polynome des Grades ≤ 2 zusammen mitder Addition von
> Polynomen (p+q)(x) :=p(x) + q(x) und der Multiplikation mit
> Konstanten (c·p)(x) := c·p(x). Wir setzen P₁(x) := x²
> + x + 1 p₂(x) := x –1 p₃(x) := x² -3x+2 Ist B (p₁,
> p₂, p₃) eine Basis von (P, +, ·)?
>  Zu  dieser Aufgabe gibt es folgende Lösung:
>  
> a₁ + a₂ + a₃= 0
>  a₂ -a₃= 0
>  a₁ -3a₂ + 2a₃= 0
>  a₁, a₂, a₃= 0   -> Die Vektoren sind Linear

> unabhängig

Wenn ich zur Überprüfung auf lineare Unabhängigkeit ansetze:
[mm] $a_1*p_1(x)+a_2*p_2(x)+a_3*p_3(x) \equiv [/mm] 0$

dann die Polynome [mm] $p_i$ [/mm] einsetze:
[mm] $a_1*(x^2+x+1)+a_2*(x-1)+a_3*(x^2-3x+2) \equiv [/mm] 0$

ausmultipliziere und nach Potenzen von x wieder ausklammere:
[mm] $(a_1+a_3)*x^2+(a_1+a_2-3*a_3)*x+(a_1-a_2+2*a_3) \equiv [/mm] 0$

kann ich einen Koeffizientenvergleich machen, da für die Nullfunktion gilt:
[mm] $0*x^2+0*x+0*1 \equiv [/mm] 0$, also egibt sich folgendes Gleichungssystem:

[mm] $a_1+a_3 [/mm] = 0$
[mm] $a_1+a_2-3a_3 [/mm] = 0$
[mm] $a_1-a_2+2a_3 [/mm] = 0$


Das ist aber verschieden von dem, was du hier oben aufgeschrieben hast.
Wie bist du darauf gekommen?

>  
> Koeffizientenvergleich
>  a₁ + a₂ + a₃= s
>  a₂ -a₃= t
>  a₁ -3a₂ + 2
>  a₃= u
>  
> a₁= s + t –(s-u)/4
>  a₂= (s-u)/4
>  a₃= (s-u)/4 –t

$p [mm] \in [/mm] P$ kann man auch schreiben als $p(x) = [mm] s*x^2+t*x+u$ [/mm] mit $s, t, u [mm] \in \IR$. [/mm]
Ist [mm] $p_1, p_2, p_3$ [/mm] eine Basis von P, so kann p dargestellt werden als
$p(x) = [mm] a_1*p_1(x)+a_2*p_2(x)+a_3*p_3(x)$ [/mm] mit [mm] $a_1, a_2, a_3 \in \IR$ [/mm]
nach ausmultiplizieren und sortieren nach Potenzen von x ergibt der
Koefizientenvergleich:
$s = [mm] a_1+a_3$ [/mm]
$t = [mm] a_1+a_2-3a_3$ [/mm]
$u = [mm] a_1-a_2+2a_3$ [/mm]

Woraus sich [mm] $a_1, a_2, a_3$ [/mm] in Abhängigkeit von s, t, u  berechnen lassen sollten.
Ist das nicht möglich oder treten Widersprüche auf, ist es keine Basis.

>  
> Antwort: Daraus folgt, dass das p1, p2, p3 keine Basis
> bilden.
>  
>
> Bisher kann ich alles soweit nachvollziehen, jedoch fehlt
> es mir an Verständnis beim Koeffizientenvergleich.
>  Warum bilden p1, p2, p3 nun keine Basis?
>  Wie müsste das Ergebnis aussehen wenn es eine Basis
> wäre?

Gruß
meili

Bezug
                
Bezug
Basis bestimmen: Frage (beantwortet)
Status: (Frage) beantwortet Status 
Datum: 09:05 Mo 11.07.2016
Autor: JXner

Danke für deine ausführliche Antwort!
Aber ein kleine Frage dazu hätte ich noch.

> Woraus sich [mm]a_1, a_2, a_3[/mm] in Abhängigkeit von s, t, u  
> berechnen lassen sollten.
> Ist das nicht möglich oder treten Widersprüche auf, ist
> es keine Basis.

D.h. wenn ich meine Gleichungen

a₁ + a₂ + a₃= s
a₂ -a₃= t
a₁ -3a₂ + 2a₃= u

nach jeweils [mm] a_1, a_2 [/mm] und [mm] a_3 [/mm] auflöse und
eine Variable nicht von s, t & u Abhängig ist, dann habe ich keine Basis?

Bezug
                        
Bezug
Basis bestimmen: Antwort
Status: (Antwort) fertig Status 
Datum: 09:13 Mo 11.07.2016
Autor: fred97


> Danke für deine ausführliche Antwort!
>  Aber ein kleine Frage dazu hätte ich noch.
>  
> > Woraus sich [mm]a_1, a_2, a_3[/mm] in Abhängigkeit von s, t, u  
> > berechnen lassen sollten.
> > Ist das nicht möglich oder treten Widersprüche auf, ist
> > es keine Basis.
>  
> D.h. wenn ich meine Gleichungen
>  
> a₁ + a₂ + a₃= s
> a₂ -a₃= t
> a₁ -3a₂ + 2a₃= u

??? Wie kommst Du auf dieses LGS ? Meilli hat Dir das richtige genannt:


$ s = [mm] a_1+a_3 [/mm] $
$ t = [mm] a_1+a_2-3a_3 [/mm] $
$ u = [mm] a_1-a_2+2a_3 [/mm] $

Zeige: dieses LGS hat für jede Wahl von s,t und u genau eine Lösung.Das bedeutet dies: jedes Polynom vom Grad [mm] \le [/mm] 2 lässt sich in eindeutiger Weise als Linearkombination von [mm] p_1,p_2 [/mm] und [mm] p_3 [/mm] schreiben. Damit ist [mm] \{p_1,p_2,p_3\} [/mm] eine Basis von P


FRED

>  
> nach jeweils [mm]a_1, a_2[/mm] und [mm]a_3[/mm] auflöse und
> eine Variable nicht von s, t & u Abhängig ist, dann habe
> ich keine Basis?


Bezug
Ansicht: [ geschachtelt ] | ^ Forum "Uni-Analysis-Sonstiges"  | ^^ Alle Foren  | ^ Forenbaum  | Materialien


^ Seitenanfang ^
ev.vorhilfe.de
[ Startseite | Mitglieder | Impressum ]